Find the general solution to the augmented matrix:
\[\begin{bmatrix} 0 & -3 & -6 & 12 \\ -3 & -3 & 6 & 27 \\ -3 & -3 & 6 & 27 \end{bmatrix}.\]Click for answer
Click for Latex code
\[ \begin{bmatrix} 0 & -3 & -6 & 12 \\ -3 & -3 & 6 & 27 \\ -3 & -3 & 6 & 27 \end{bmatrix} \]Solution:
\begin{solution} Reduced Row Echelon Form: \[\begin{bmatrix} 1 & 0 & -4 & -5 \\ 0 & 1 & 2 & -4 \\ 0 & 0 & 0 & 0 \end{bmatrix}\] General Solution: \[ \begin{bmatrix} x_{ 1 } \\ x_{ 2 } \\ x_{ 3 } \\ \end{bmatrix} = \qquad \begin{bmatrix} -5 \\ -4 \\ 0 \end{bmatrix} \qquad + x_3 \begin{bmatrix} 4 \\ -2 \\ 1 \end{bmatrix} \] \end{solution}